It can be inferred from the passage that the author would be LEAST likely to agree with which one of the following?

amarachicynthia on February 6, 2021

Answer choice E

Why is E incorrect?

Reply
Create a free account to read and take part in forum discussions.

Already have an account? log in

SamA on February 6, 2021

Hello @amarachicynthia,

I understand why E is confusing because the author does include quite a bit of information about prion reproduction in the third paragraph. However, I think the author would still agree that this is not a complete understanding. It is a tough question if you only look to the third paragraph, but the last sentence of the passage reveals the answer.

Line 58: "... the exact mechanishs by which prions reproduce themselves and cause cellular destruction have yet to be completely understood."

It is pretty clear that the author would agree with answer choice E, which is why we can eliminate it.

Sometimes the last sentence of a passage will have important information, so don't forget it.